You are on page 1of 107

11

GENERAL
MATHEMATICS
Self-Learning Module
The Self-Learning Module was collaboratively evaluated
and reviewed by the Learning Resource Evaluator/s of the
Schools Division of Lucena City. We encourage teachers and
other educational stakeholders to e-mail their feedbacks,
comments and recommendations to
lucena.city@deped.gov.ph.

Department of Education
Schools Division of Lucena City
General Mathematics-Grade 11
Self-Learning Module

Republic Act 8293, section 176 states that: No Copyright shall subsist in any work of
the Government of the Philippines. However, prior approval of the government agency or office
wherein the work is created shall be necessary for exploitation of such work to profit. Such agency or
office may, among other things, impose as condition the payment of royalties.

Borrowed materials (i.e. songs, stories, poems, pictures, photos, brand names, trademarks,
etc.) included in this Mathematics 11: General Mathematics Self-Learning Module are owned by their
respective copyright holders. DepEd is represented by the Filipinas Copyright Licensing Society
(FILCOLS), Inc. in seeking permission to use these materials from their respective copyright owners.
All means have been exhausted in seeking permission to use these materials. The publisher and
authors do not represent nor claim ownership over them.

Only institution and companies which have entered an agreement with FILCOLS and only
within the agreed framework may copy from these module Those who have not entered the
agreement with FILCOLS must, if they wish to copy, contact the publishers and authors directly.

Authors and publishers may email or contact FILCOLS at filcols@gmail.com or (02) 439-2204,
respectively.
Development Team of the Self-Learning Module
Writers: Arvin A. Asnan, Azalea A. Gallano, Azenith G. Mercado, Dennis E.
Ibarrola,
Rey Mark R. Queaño, Angelo S. Villanueva, January B. Regio,
Mary Grace D. Constantino, Jea Aireen Charimae M. De Mesa, Ann Michelle M. Jolo,
Raiza Ann Ebarle.

Illustrator: Hanna Lorraine G. Luna

Editors: Dr. Anicia J. Villaruel and Roy O. Natividad

Layout Artist: Sayre M. Dialola

Management Team: Dr. Wilfredo E. Cabral, Job S. Zape Jr., Eugenio S. Adrao,
Fe M. Ong-Ongowan, Dr. Hermogenes M. Panganiban, Dr. Babylyn M. Pambid,
Josephine T. Natividad, Dexter M. Valle, Dr. Anicia J. Villaruel, Sayre M. Dialola and

Inilimbag sa

DepEd Region IV-A CALABARZON


Office Address: Gate 2, Karangalan Village, Cainta Rizal
Philippines 1800
Telephone: (647) 4914 local 110
E-mail Address: lrmd.calabarzon@deped.gov.ph
PREFACE

This module is intended for Senior High School learners who opt to undergo
Alternative Delivery Mode (ADM) of learning. With the clientele in mind, each topic
has been presented in a level which the target users can comprehend. Localization
is considered as well for learners to be able to easily relate with the discussion.
Nevertheless, this module may also be utilized in regular Senior High School
Mathematics classes as lessons were organized in accordance to the Curriculum
Guide for General Mathematics including Functions, Business Mathematics, and
Logic.
Every lesson starts with a pretest. Activities prompting learners’ prior
knowledge and encouraging collaboration among others are followed by questions
requiring critical thinking skills. Discussions of the lessons are further clarified
through various examples to strengthen the learning process of the learners.
Exercises through problem solving are also provided. At the end of each lesson, a
posttest is given to evaluate whether the objectives have been met by the learners.
Creative and reflective thinking skills of learners are further enhanced as they are
required to write and reflect as how to apply in real-life the values integrated in a
topic. Definitely, this module aims to make every Batang CALABARZON a life-long
Mathematics learner.

The Writers

iii
TABLE OF CONTENTS
Preface …………………………………………………………………………….. iii
Table of Contents………………………………………………………………..... iv

Chapter I – Functions
Lesson 1: Representing Functions ………………………………………….
2
Lesson 2: Evaluating Functions & Solving Word Problems ………………
13
Lesson 3: Operations on Functions …………………………………………
19

Chapter II – Rational Functions


Lesson 4: Rational Function, Equation & Inequality ……..……………….
26
Lesson 5: Rational Function …………..…………………………………….. 37
Lesson 6: Representations of Rational Function ….………………………
49
Lesson 7: Graphing of Rational Function ….……………………………….
54

Chapter III – One-to-One Functions


Lesson 8: One-to-One Function …..…………………………………….. 61
Lesson 9: Inverse Function …..……………………………………………… 67
Lesson 10: Domain and Range of Inverse Function …..…………………..
72
Lesson 11: Graphs of Inverse Function ……..………………………………
77
Lesson 12: Problems Involving Inverse Function …..………………………
84

iv
“The functions of the family in a highly differentiated society
are not to be interpreted as functions directly on behalf of
the society, but on behalf of personality.”
——————- Talcott Parsons ———————

LESSON 1: Representing Functions


LESSON 2: Evaluating Functions and Solving Word
Problems
LESSON 3: Operations on Functions
CHAPTER I-FUNCTIONS
Lesson 1: Representing Functions
--------------------------------------------------------------------------------------------------------------
Introduction
In this topic, learners are expected to demonstrate understanding the key
concepts of functions specifically on representing functions in real life situations,
evaluating and solving operations on functions.

Directions for the User


A pretest is prepared for you to assess which part of the lesson you need to
give more focus on. After answering it, you may then proceed to the Self-Learning
Module proper consisting of the Procedure/Learning Experience and Reflection.
Before leaving every lesson, you will also answer a posttest. Ready? Begin!
--------------------------------------------------------------------------------------------------------------
Pretest
Read and analyze each of the following then shade the circle that corresponds
to the letter of the best answer.
A B C D
O O O O 1. It is a relation where each element in the domain is related to only
one value in the range by some rule.
A. Function C. Domain
B. Range D. Piecewise

O O O O 2. Which of the following relations is a function?


A. X = {(1,2), (3,4), (1,7), (5,1)}
B. G = {(3,2), (2,1), (8,2), (3,7)}
C. H = {(4,1), (2,3), (2, 6), (7, 2)}
D. Y = {(2,9), (3,4), (9,2), (6,7)}

O O O O 3. A person is earning Php 500 per day for doing a certain job. Which
of the following expresses the total salary S as function of the
number n of days that the person works?
A. S(n) = 500 + n C. S(n) = 500n
B. S(n) = 500/n D. S(n) = 500 - n

2
O O O O 4. A jeepney ride in Lucena costs Php 9.00 for the first 4 kilometers
and each additional kilometer adds Php 2.00 to the fare. Use a
piecewise function to represent the jeepney fare F in terms of the
distance d in kilometers.
A. F(d) = 9.00 if 0 > d ≤ 4, (9 + 2.00[n]) if d < 4
B. F(d) = 9.00 if 0 < d < 4, (9 + 2.00) if d > 4
C. F(d) = 9.00 if 0 > d ≤ 4, (9 + 2.00) if d < 4
D. F(d) = 9.00 if 0 < d ≤ 4, (9 + 2.00[n]) if d > 4

O O O O 5. Determine the domain and range of the function 3x - y = 4.


A. D = {x/x є Ɽ} and R = {y/y є Ɽ}
B. D = {x/x > 3} and R = {y/y є Ɽ}
C. D = {x/x є Ɽ} and R = {y/y < 4}
D. D = {x/x > 3} and R = {y/y < 4}
--------------------------------------------------------------------------------------------------------------
Competency
Represents real-life situations using functions, including piece-wise
functions. M11GM-Ia-1

Objectives
At the end of the lesson, the learners are expected to:
1. Understand and classify functions.
2. Represent real-life situations using functions, including piece-wise
function.
3. Provide illustrations or pictures showing functions.

Procedure/Learning Experience
Activity

Group A Group B

Group the following given relations as A or B by writing the letter to the


corresponding column. Then justify your way of grouping them.
Reason for Grouping:
—————————————————————————————————————

3
Given:
A. J = {(2,1), (3,2), (4,3), (5,4)} B.K = {(-2,4), (-1,1), (-2,0), (1,5)}

X -3 -2 -1 -1 0 x 1 2 3 4 5
C. D.
y 5 4 3 2 1 y ±1 ±4 ±9 ±16 ±25

E. F.
Japan -3
Manila 1 -2
Bangkok Indonesia
-1
Jakarta Philippines 4
1
9 2

G. f(x) = y + 2 H. r2 = x2 + y2
I. J.

Analysis
1. Given the examples of relation in the activity, how will you define
relations?
2. What did you consider in grouping the relations as A or B?
3. What can you say about the relations in Group A? in Group B?
4. From the groupings you have done, which relations show a function? Why?
5. How will you define a function?

Abstraction
A relation is any set of ordered pairs. The set of all first components of the
ordered pairs is called the domain of the relation, and the set of all second
components is called the range.
A function is a relation or rule of correspondence such that each element in
the domain corresponds to exactly one element in the range.

4
Example 1: Which of the following relations are functions?
A = {(1,2), (2,3), (3,4), (4,5)}
B = {(3,3), (4,4), (5,5), (6,6)}
C = {(1,0), (0, 1, (-1,0), (0,-1)}
D = {(a,b), (b, c), (c,d), (a,d)}
Solution:
The relations A and B are functions because each element in the domain
corresponds to a unique element in the range. Meanwhile, relations C and D are
not functions because they contain ordered pairs with the same domain
[C = (0,1) and (0,-1), D = (a,b) and (a,d)].

Example 2: Given some real-life relationships, identify those which are


functions
and those which are mere relations.
A. The rule which assigns to each person the name of his aunt.
B. The rule which assigns to each person the name of his father.
C. The rule which assigns a chemical element to its chemical symbol.
D. The rule which assigns to each cellular phone unit to its phone
number.
E. The rule which assigns a capital city to each province.

Solution:
A. There are persons who have more than one aunt. Thus, the rule
which assigns to each person the name of his aunt is mere a
relation.
B. Every person has one father. Thus, the rule which assigns to each
person the name of his father is a function.
C. To each chemical element, a unique chemical symbol is assigned.
No chemical element has more than one chemical symbol. Thus,
this relationship is a function.
D. There are cellular phone owners who use more than one SIM
card. Therefore, the rule which assigns to each cellular phone unit
its phone number is a mere relation.
E. Each province has one capital city. Thus, the rule for assigning a
capital city to each province is a function.

5
Example 3: Which of the following mapping diagrams represent functions?
Domain Range
A.
a x
b y
c

B. x a

y b
c

C.
Jana Ken

Dona Mark
Maya Rey

Solution:
The relations A and C are functions because each element in the domain
corresponds to a unique element in the range. However, B is a mere relation/not
function because there is a domain which corresponds to more than one range.

A relation between two sets of numbers can be illustrated by graph in


the Cartesian Plane, and that a function passes the vertical line test.
A graph of a relation is a function if any vertical line drawn passing
through the graph intersects it at exactly one point.

Vertical Line Test

6
Example 4: Which of the following can be the graphs of function?
A. B.

C. D.

Solution:
A and B are graphs of functions while C and D are not because they do not
pass the vertical line test.

Domain and Range of Functions

The definition of function does not allow an element of its domain to


have two or more corresponding elements in the range. The definition
however, does not restrict two or more elements in the domain to
correspond to the same element in the range.
If the domain of a function is not stated and restricted, it is understood
that the domain is the set of all real numbers, Ɽ which makes the values of
the function real. (Orines, 2016)

7
Example 5: Determine the domain and range of each of the following
functions.
A. y = x C. y =

B. y = x2 D. y =
Solution:
For y = x, the independent variable x can take any value in the set of real
numbers which makes the values of the dependent variable y real. Hence,
the domain of y is the set of real numbers and its range is also the set of
real numbers. In set notation:

D = {x/x є Ɽ} R = {y/y є Ɽ}
For y = x2, its domain is the set of real numbers. Note that for any real
number x, x2 ≥ 0. Hence, the range of y is the set of real numbers greater
than or equal to 0. In set notation:

D = {x/x є Ɽ} R = {y/y ≥ 0}
For y = , the domain is restricted to only those values of x that make y
real, that is, x ≠ 0. Since the denominator x cannot take a value of 0, y ≠ 0.
In set notation:
D = {x/x ≠ 0} R = {y/y ≠ 0}
For y = , the domain is restricted to only those values of x that make y
nonnegative real numbers. The domain and range respectively are:
D = {x/x ≥ 0} R = {y/y ≥ 0}

Functions can often be used to model real-life situations. Identifying an


appropriate functional model will lead to a better understanding of various
phenomena.

Functions as representations of real-life situations

8
Example 6:
A. Give a function B that can represent the amount of battery charge of a
cellular phone in h hour, if 12% of battery was lost every hour.
B. Contaminated water is subjected to a cleaning process. The concentration
of the pollutants is initially 5 mg per liter of water. If the cleaning process
can reduce the pollutant by 10% each hour, define a function that can
represent the concentration of pollutants in the water in terms of the
number of hours that the cleaning process has taken place.

C. Squares of side x are cut from each corner of a 10 in x 8 in rectangle, so


that its sides can be folded to make a box with no top. Define a function in
terms of x that can represent the volume of the box.
Solution:
A. Since every hour losses 12% of the battery, then the amount of battery
function is B (h) = 100 - 0.12h
B. After 1 hour, the concentration of pollutants is (5)*(0.90). After 2 hours, it is
this value times 0.90 or [(5)*(0.90)(0.90 = 5(0.90)2 . In general, after t
hours, the concentration is C(t) = (5)(0.90)t mg per liter of water.
C. The length and width of the box are 10 - 2x and 8 - 2x, respectively. Its
height is x. Thus, the volume of the box can be represented by the function
V(x) = (10 - 2x)(8 - 2x)x = 80x - 36x2 + 4x3.

Piecewise Functions
These functions are defined compositely using several expressions and
different interval domains.

Example 7:
A. A user is charged Php 250.00 monthly for a particular mobile plan, which
includes 200 free text messages. Messages in excess of 200 are charged
Php 1.00 each. Represent the monthly cost for text messaging using the
function t(m), where m is the number of messages sent in a month.
B. A certain chocolate bar costs Php 50.00 per piece. However, if you buy
more than 5 pieces they will mark down to a price of Php 48.00 per piece.
Use a piecewise function to represent the cost in terms of the number of
chocolate bars bought.
Solution:
A. t(m) = {250 if 0 < m ≤ 200, (250 + m]) if m < 200

9
Application
A. Determine whether the given relation is a function or a mere relation.
1. f(x) = {(0,5), (5, 0), (-5,0), (0,-5)}
2. t(x) = {(m,b), (u, k), (k,m), (u,b)}
3. the rule which assigns to each person the name of his mother
Domain Range
4.

5.

B. Determine the domain and range of each function.


1. y = 2x - 1
2. h(x) =
2
3. m(x) = x + 3

4. y =
5. y = |x| + 2

C. Represent the following situations as function.


1. Arvin is earning Php 750.00 per day to do a certain job. Express the total
salary S as a function of the number n of days that he works.
2. A computer shop charges Php 15.00 in every hour of computer rental.
Represent your computer rental fee using the function R(t) where t is the
number of hours you spent on the computer.
3. A school fair committee wants to sell t-shirts for their school fair. The
committee found a supplier that sells t-shirts at a price of Php 250.00 per
piece. Express the total amount A as a function of the number n of t-shirts
that the school ordered from the supplier.

10
Reflection
This lesson helps you clearly understand the concept of functions by using
real-life situations. You learned the concepts of domain and range as well as how
piecewise functions differ from the other types of functions. Can you think of some
other real-life situations that show functions?
————————————————————————————————————
Posttest
Read and analyze each of the following questions then shade the circle that
corresponds to the letter of the best answer.
A B C D
O O O O 1. Which of the following relations is a function?
A. X = {(-1,2), (-3,4), (-1,7), (5,1)}
B. G = {(-3,2), (3,1), (-3,2), (5,7)}
C. H = {(6,1), (-2,3), (2, 6), (7, 2)}
D. Y = {(2,3), (3,2), (-2,3), (3,-2)}

O O O O 2. Which of the following relations is a function?


A. The rule which assigns to each person the name of his
brother.
B. The rule which assigns the name of teachers you have.
C. The rule which assigns to each cellular phone unit its
phone number.
D. The rule which assigns each person a surname.

O O O O 3. Determine the domain and range of the function f(x) = 5x2 + 2.


A. D = {x/x є Ɽ} and R = {y/y ≥ 2}
B. D = {x/x > 2} and R = {y/y є Ɽ}
C. D = {x/x є Ɽ} and R = {y/y < 2}
D. D = {x/x > 5} and R = {y/y < 2}

11
O O O O 4. A person can encode 1000 words in every hour of typing job.
Which of the following expresses the total words W as a
function of the number n of hours that the person can encode?
A. W(n) = 1000 + n C. W(n) = 1000n
B. W(n) = 1000/n D. W(n) = 1000 - n

O O O O 5. A tricycle ride costs Php 10.00 for the first 2 kilometers, and
each additional kilometer adds Php 5.00 to the fare. Use a
piecewise function to represent the taxi fare in terms of the
distance d in kilometers.
A. F(d) = {10.00 if 0 > d ≤ 2, (10 + 5.00[n]) if d < 2
B. F(d) = {10.00 if 0 < d < 2, (10 + 5.00) if d > 2
C. F(d) = {10.00 if 0 > d ≤ 2, (10 + 5.00) if d < 2
D. F(d) = {10.00 if 0 < d ≤ 2, (10 + 5.00[n]) if d > 2
-----------------------------------------------------------------------------------------------------------
End of Lesson 1

12
Lesson 2: Evaluating Functions and Solving Word Problems
----------------------------------------------------------------------------------------------------
Pretest
Read and analyze each of the following questions, then shade the circle that
corresponds to the letter of the best answer.
A B C D
O O O O 1. Given g(x) = , determine g(5).
A. 11 C. -11
B. 7/2 D. -7/2

O O O O 2. For what values of x can we not evaluate the function


f(x) = ?

A. ±1 C. ±3
B. ±2 D. ±4

O O O O 3. Find the function value given h(x) = 9 - 5x of x = 3m.


A. 9 - 15m C. 9 - 15m2
B. 9 + 15m D. 9 + 15m2

For numbers 4-5.


A proposed tricycle fare would charge Php 20.00 for the first 5 km of travel
and Php 0.75 for each additional kilometer over the proposed fare.

O O O O 4. Find the fare function f(x) where x represents the number of


kilometers traveled.
A. f(x) = Php 20.00 + 0.75x C. f(x) = Php 16.25 + 0.75x
B. f(x) = Php 20.00 - 0.75x D. f(x) = Php 16.25 - 0.75x

O O O O 5. Find the proposed fare for a distance of 55 km.


A. Php 57.50 C. Php 60.50
B. Php 91.25 D. Php 105.75
------------------------------------------------------------------------------------------------------------

13
Competencies
1. Evaluates a function.M11GM-Ia-2
2. Solves problems involving functions.M11GM-Ia-4

Objectives
At the end of the lesson, the learners are expected to:
1. Evaluate functions.
2. Understand and analyze problems involving functions.
3. Solve problems involving functions.

Procedure/Learning Experience
Activity
Given the following equations, find the value of y if x = 3.
1. y = x - 9
2. y = 3x + 7
3. y = x2 + 4x - 10
4. y = 2x2 - 6x + 26
5. 3y = 3x2 – 6

Analysis
1. In the activity, how did you arrive with the value of y?
2. What skills are needed in order to arrive with the answer?
3. What does it mean to evaluate an equation?
4. Does evaluating an equation the same as evaluating a function? Why?

Abstraction
When a function f is defined by y = f(x), then x is called the independent
variable. Any element of the domain can be chosen as a value of the independent
variable. The output or the resulting value determines the corresponding value of
the dependent variable y.
The value of the dependent variable y = f(x) that corresponds to a specific
value of an independent variable x is found by substituting x into the function rule,
then, simplifying the resulting expressions. Such process is called evaluating a

Rule in Evaluating a Function


Replace every x in the function rule f(x) with the given replacement
value, then, simplify the resulting numerical expression.

14
Example 1: Evaluate the function for each replacement
value of x.
A. x = 1 C. x = 0
B. x = -2 D. x = 3
Solution:
To evaluate the given function, replace x in f(x) with the given replacement
value then, simplify the results.
If x = 1, then

If x = -2, then

If x = 0, then

If x = 3, then

In solving problems involving functions, George Polya’s 4-step rule can


facilitate its solution. First, explore the problem carefully and choose a varia-
ble for an unknown quantity. Then, plan the solution. Write equations that
describe the relationship. Next, solve the equations and other number rela-
tions, determine the required quantities, and answer the questions in the
problem. Finally, check the solution using other approaches and
examine the appropriateness of the answers (Orines, 2016).

Problems Involving Functions


Example 2. Lucena Network charges Php 450 monthly cable connection fee
plus Php 130 for each hour of Pay-Per-View (PPV) event regardless of a
full hour or a fraction of an hour.

15
a. Find payment f(x) where x represents the number of PPV hours.
b. What is the monthly bill of a customer who watched 25 hours of PPV
events?
c. What is the monthly bill of a customer who watched 0.5 hour of PPV
events?
Solution:
a. Php 450.00 is fixed monthly cable connection fee.
Let x = number of PPV hours in a month
Php 130(x) = amount of PPV payment in a specific hour
The payment function is f(x) = Php 450.00 + Php 130.00(x).

b. The monthly bill of a customer who watched 25 hours PPV events can
be represented by 24 < x ≤ 25.
f(x) = Php 450.00 + Php 130.00(x)
f(25) = Php 450.00 + Php 130.00(25)
= Php 450.00 + Php 3,250.00
= Php 3,700.00

c. The monthly bill of a customer who watched 0.5 hour PPV events
can be represented by 0 < x ≤ 1.
f(x) = Php 450.00 + Php 130.00(x)
f(25) = Php 450.00 + Php 130.00(1)
= Php 450.00 + Php 130.00
= Php 580.00

Example 4. A proposed train fare would charge Php 50.00 for the first 10 km
or
less of travel and Php 1.25 for each additional kilometer over the
proposed fare.
a. Find the fare function f(x) where x represents the number of
kilometers traveled.
b. Find the proposed fare for a distance of 11 km.
c. Find the proposed fare for a distance of 35 km.
Solution:
a. f(x) = 50 for 0 < x ≤ 10
Let x = number of kilometers traveled
x - 10 = number of kilometers traveled over and above 10 km.
f(x) = 50 + 1.25(x - 10)
f(x) = 50 + 1.25x - 12.5

16
b. f(x) = 37.50 + 1.25x
f(11) = 37.50 + 1.25(11)
f(11) = 37.50 + 13.75
f(11) = 51.25

c. f(x) = 37.50 + 1.25x


f(35) = 37.50 + 1.25(35)
f(35) = 37.50 + 43.75
f(35) = 81.25

Application
A. Evaluate the following functions at x = 5.
1. f(x) = x - 5
2. g(x) = x2 - 4x + 7
3. h(x) =

4.
5. f(x) = |3x - 9|

B. Find the function value.


1. If f(x) = 7x - 5, find f(4) and f(-5)
2. If f(x) = x2 + 3x - 9, find f(1) and f(-3)
3. If f(x) = , find f(3) and f(-2)

C. Solve this problem: “A mall charges Php 35.00 for the first hour or a fraction of
an hour as parking fee. An additional Php 15.00 is charged for every
additional hour, or a fraction of an hour of parking. The parking area operates
from 7 AM to 12 midnight everyday.”
a. Write a function rule for the problem.
b. How much will be charged to a car owner who parked for 8 hours?
c. How much will be charged to a car owner who parked a car from 8:00
AM to 1:30 PM?

Reflection
This lesson teaches you how to evaluate a given function. With this lesson,
you also learn the real-life application of function and how problems will be
solved. What insight/s have you gained as you go along with the procedure of

17
Post-Test
Read and analyze each of the following then shade the circle that corre-
sponds to the letter of the best answer.
A B C D
O O O O 1. Given g(x) = , determine h(4).
A. 7 C. -7
B. 13/3 D. -13/3

O O O O 2. Find the value of f(x) = │3x│- 2 at x = -5.


A. 17 C. –17
B. 13 D. –13

O O O O 3. Find the function value given h(x) = 3x - 8 of x = 9a +1.


A. 27a + 5 C. 18a + 11
B. 27a - 5 D. 18a - 11

For numbers 4-5.


Mark charges Php 100.00 for an encoding work. In addition, he charges
Php 5.00 per page of printed output.

O O O O 4. Find a function f(x) where x represents the number page of printed


out.
A. f(x) = Php 100.00 + 5x C. f(x) = Php 100x + 5
B. f(x) = Php 100 - 5x D. f(x) = Php 100x – 5

O O O O 5. How much will Mark charge for a 55-page encoding and printing
work?
A. Php 275 C. Php 375
B. Php 175 D. Php 475
-----------------------------------------------------------------------------------------------------------
End of Lesson 2

18
Lesson 3: Operations on Functions
-------------------------------------------------------------------------------------------------------------
Pretest
Perform each of the following then shade the circle that corresponds to the
letter of the best answer.
Let and
A B C D
O O O O 1. Find
A. 2x2 - 3x + 3 C. 2x2 + 3x + 3
2
B. 2x + 3x - 3 D. 2x2 - 3x - 3

O O O O 2. Find
A. 2x2 - 3x + 17 C. 2x2 + 3x + 17
B. 2x2 + 3x - 17 D. 2x2 - 3x - 17

O O O O 3. Find
A. 6x3 + 20x2 - 21x - 70 C. 6x3 - 20x2 - 21x - 70
B. 6x3 + 20x2 + 21x + 70 D. 6x3 - 20x2 + 21x + 70

O O O O 4. Find

A. C.

B. D.

O O O O 5. Find
A. C.
B. D.
-------------------------------------------------------------------------------------------------------------

19
Competency
Performs addition, subtraction, multiplication, division, and composition of
functions. M11GM-Ia-3

Objectives
At the end of the lesson, the learners are expected to:
1. Perform operations & composition of functions.
2. Determine the domain of the functions.

Procedure/Learning Experience
Activity
Perform the following operations on algebraic expressions.
1. Find the sum of 3x2 - 4x + 5 and 6x2 +7x - 9

2. Add: + .

3. Find the difference of and .

4. Find the product of and .

5. Divide by and .

Analysis
1. What have you realized while doing the activity?
2. What rules have you applied in adding and subtracting algebraic
expressions?
3. What did you do in multiplying and dividing algebraic expressions?
4. Do you think performing operations on algebraic expression is the
same as performing operations in functions? How do you say so?

Abstraction
Just as two real numbers can be combined by the operations of addition,
subtraction, multiplication and division to form real numbers, two functions can be
combined to create a new one (Alferez, Duro & Tupaz, 2008).
For example, and , one can form the sum,
difference, product and quotient as follows.
Sum
Difference
Product

20
The Sum of Functions
If f and g are functions:
Their sum f + g is the function defined by: (f + g)(x) = f(x) + g(x)
The domain of f + g consists of the numbers x that are in the domain

Example 1: Let and . Find .


Determine the domain of .
Solution:

Domain: {x/x є Ɽ}

The Difference of Functions


If f and g are functions:
Their difference f - g is the function defined by: (f - g)(x) = f(x) - g(x)
The domain of f - g consists of the numbers x that are in the domain of f
and in the domain of g.

Example 2: Let and . Find .


Determine the domain of .

Solution:

Domain: {x/x є Ɽ}

21
The Product of Functions
If f and g are functions:
Their product f · g is the function defined by: (f · g)(x) = f(x) · g(x)
The domain of f · g consists of the numbers x that are in the domain of f

Example 3: Let and . Find .


Determine the domain of .
Solution:

The Quotient of Functions


If f and g are functions:
Their quotient f / g is the function defined by: =
The domain of f / g consists of the numbers x that are in the domain of

Domain: {x/x ≠ ±1, x ≠ }

Example 4: Let and . Find .


Determine the domain of .
Solution: =

Domain: {x/x є Ɽ}

22
The Composite Functions
Given two functions f and g, the composite function is defined as .
The domain of is the set of all numbers x in the domain of g such that
g(x) is in the domain of f.

The symbol is read as “f circle g”. The idea suggests that in computing
for , first apply the function g to x and then the function f to g(x).

Example: Let and . Find the following and deter-


mine the domain.
a. b.

Solution:
a. To find , replace all x’s in g(x) with x - 1.

Domain: {x/x є Ɽ}

b.

Domain: {x/x є Ɽ}

Application
A. Let and . Find the following:
1.
2.
3.
4.
5.

23
B. Define the following functions and determine the domain of the results.
1. Find:
Given: and

2. Find:
Given: and

3. Find:
Given: and

4. Find:
Given: and

5. Find:
Given: and

Reflection
This lesson teaches you how to perform the operation in defining and solving
functions. You have realized that performing operations on functions is the same
as solving algebraic expressions.
In this lesson, you have also realized the importance of mastering every skill
in Mathematics since these are all interrelated and prerequisite to other skills. Do
you experience solving any real-life problem that requires step-by-step solution?
--------------------------------------------------------------------------------------------------

24
Posttest
Perform each of the following then shade the circle that corresponds to the
letter of the best answer.
Let and
A B C D
O O O O 1. Find
A. 5x2 - x + 8 C. 5x2 + x - 8
B. 5x2 + x + 8 D. 5x2 - x - 8

O O O O 2. Find
A. 5x2 - 3x + 8 C. 5x2 - 3x - 8
B. 5x2 + 3x + 8 D. 5x2 + 3x - 8

O O O O 3. Find
A. -5x3 + 38x2 + 16x C. -5x2 + 38x + 16
B. 5x3 - 38x2 - 16x D. 5x2 - 38x - 16

O O O O 4. Find

A. C.

B. D.

O O O O 5. Find
A. 5x2 - 18x + 80 C. 5x2 - 82x + 336
B. 5x2 - 18x - 80 D. 5x2 + 82x - 336
----------------------------------------------------------------------------------------------------
End of Lesson 3

25
“A man has free choice to the extent that he is rational”
——————- Thomas Aquinas ———————

LESSON 4: Rational Functions, Equations, and


Inequalities
LESSON 5: Rational Functions
LESSON 6: Representations of Rational Functions
LESSON 7: Graphing Rational Functions
CHAPTER II – RATIONAL FUNCTIONS
Lesson 4: Rational Functions, Equations and Inequalities
-------------------------------------------------------------------------------------------------------------
Introduction
In this topic, the learners are expected to demonstrate understanding the key
concepts of functions specifically on representing real-life situations using rational
functions, classifying rational functions, equations and inequalities, solving rational
equations and inequalities and representing a rational function by its: (a) table of
values, (b) graph, and (c) equations.

Directions for the User


A pretest is prepared for you to assess which part of the lesson you need to
give more focus on. After answering it, you may then proceed to the Self-Learning
Module proper consisting of the Procedure/Learning Experience and Reflection.
Before leaving this module, you will also answer a posttest. Ready? Begin!
------------------------------------------------------------------------------------------------------------
Pretest
Read and analyze each of the following questions then shade the circle that
corresponds to the letter of the best answer.
A B C D
O O O O 1. Which of the following is not a rational function?
A. C.
B. D.

O O O O 2. Which of the following is a rational equation?


A. C.

B. D.

O O O O 3. Which of the following is a rational inequality?


A. C.

B. D.

27
O O O O 4. Solve for x:
A. x = 4 C. x = -4
B. x = -5 D. x = 5
O O O O 5. Solve for x:
A. [1,4] C. [1,4)
B. (1,4] D. (1,4)
-------------------------------------------------------------------------------------------------------------
Competencies
1. Distinguishes rational function, rational equation, and rational
inequality. M11GM-Ib-2
2. Solves rational equations and inequalities.M11GM-Ib-3

Objectives
At the end of the lesson, the learners are expected to:
1. Define and differentiate rational function, rational equation and rational
inequality.
2. Provide illustrations of rational function, rational equation, and rational
inequality.
3. Solve rational equations and inequalities.

Procedure/Learning Experience
Activity
Perform the following operations on fractions.
1. Find the sum of , , and .
2. Add: + .
3. Find the difference of and .
4. Find the product of , and .
5. Divide by .
Analysis
1. What have you realized while doing the activity?
2. What rules have you applied in adding and subtracting fractions?
3. What did you do in multiplying fractions?
4. What did you do in dividing fractions?
5. Do you think the same rules will be applied if we are solving polynomials in a
form of fractions? How do you say so?
6. Give examples of polynomials in a form of fraction.

28
Abstraction
A rational expression is a fraction in which the numerator and/or the
denominator are polynomials. Here are some examples of rational expressions.
, , ,

Rational Function
A rational function r(x) is a function in the form of r(x) = , where p(x)
and q(x) are polynomial functions, and q(x) ≠ 0.

Example 1 : Here are some examples of rational functions.


a. where x ≠ -1

b. where x ≠ 1, -1/3
Rational Equation

Example 2 : Here are some examples of rational equations.


a.

b.
Rational Inequality
It is an inequality that contains rational expressions.

Example 3 : Here are some examples of rational inequalities.

a.

b.

29
Solving Rational Equations

To solve rational equations:


Eliminate all fractions. When solving rational equations, multiply
the entire problem by the least common denominator or LCD.
Simplify the resulting equation. To simplify the equation, you may
need to distribute and combine like terms.
Solve the simplified equation. Equate the simplified equation to ze-
ro then find the value/s of the unknown variable.
Check each solution. Substitute each solution into original equa-

Example 1: Solve:

Step 1: Eliminate all fractions. In


this case, we need to multiply by
the LCD to eliminate the fractions.

Step 2: Simplify the resulting


equation. To simplify the equation
you may need to distribute and
combine like terms.

Step 3: Solve the simplified


equation. In this case, we need to
get the x’s on one side and the
numbers of the other side.

Step 4: Check each solution. In


this case, the only number that
would make the problem
undefined is 0. Since our answer is
not 0, the answer is accepted.

30
Example 2: Solve:

Step 1: Eliminate all fractions.


In this case, we need to multiply
by the LCD to eliminate the
fractions.

Step 2: Simplify the resulting


equation. To simplify the
equation you may need to
distribute and combine like
terms.

Step 3: Solve the simplified


equation. In this case, we need 5x - 4x = 12 + 15
to get the x’s on one side and
the numbers of the x = 25
other side.

Step 4: Check each solution.


In this case, the only numbers
that would make the problem un-
x = 25
defined are 3 or –3. Since our
answer is not 3 or –3, the an-
swer is accepted.

31
Solving Rational Inequalities

To solve rational inequalities, these are the following steps:


1. Write the inequality in the correct form. One side must be zero
and the other side can have only one fraction. Simplify the fractions if
there is more than one fraction.

2. Find the key or critical values. To find the key/critical values, set
the numerator and denominator of the fraction equal to zero and
solve.

3. Make a sign analysis chart. To make a sign analysis chart, use


the key/critical values found in Step 2 to divide the number line into
sections.

4. Perform the sign analysis. To do the sign analysis, pick one


number from each of the sections created in Step 3 and plug that
number into the polynomial to determine the sign of the resulting an-
swer. The sign of this answer (positive or negative) will be sign of the
entire section. You can check different number from the same section
if you want to verify your answer.

5. Use the sign analysis chart to determine which sections satisfy


the inequality. If the inequality is less than zero or less than or
equal to zero, then you want all of the negative sections found in the
sign analysis chart. If the inequality is greater than zero or greater
than or equal to zero, then you want all of the positive sections found
in the sign analysis chart.

32
Example 1: Graph:

Step 1: Write the inequality in the


correct form. One side must be
zero and the other side can have
only one fraction, so simplify the
fractions if there is more than one
fraction.
Step 2: Find the key or critical
values. To find the key/critical
values, set the numerator and
denominator of the fraction equal to
zero and solve.
Step 3: Make a sign analysis
chart. To make a sign analysis
chart, use the key/critical values
found in Step 2 to divide the num-
ber line into
sections.
Step 4: Perform the sign analy-
sis. To do the sign analysis, pick
one number from each of the sec-
tions created in Step 3 and plug
that number into the polynomial to
determine the sign of the resulting
answer. In this case, you can
choose x = –5 which results in
–1.333, x = 0 which results in +12,
x = 2 which results in –6, and x = 4
which results in +2.666.
Step 5: Use the sign analysis
chart to determine which sections
satisfy the inequality. In this case,
we have less than or equal to zero,
so we want all of the negative sec-
tions. Notice that x ≠ 1 because it
would make the original problem
undefined, so you should use an
open circle at x = 1 instead of a
closed circle to draw the graph.

Step 6: Use interval notation to


write the final answer.

33
Example 2: Graph:

Step 1: Write the inequality in the


correct form. One side must be
zero and the other side can have only
one fraction, so simplify the fractions if
there is more than one fraction.
Step 2: Find the key or critical
values. To find the key/critical
values, set the numerator and
denominator of the fraction equal to
zero and solve.
Step 3: Make a sign analysis chart.
To make a sign analysis chart, use the
key/critical values found in Step 2 to
divide the
number line into sections.
Step 4: Perform the sign analysis.
To do the sign analysis, pick one num-
ber from each of the sections created
in Step 3 and plug that number into
the polynomial to
determine the sign of the resulting
answer. In this case, you can choose
x = –4 which results in
–0.025, x = 0 which results in +0.75, x
= 2 which results in –2.5, and x = 4
which results in +2.
Step 5: Use the sign analysis chart
to determine which sections satisfy
the inequality. In this case, we have
greater than or equal to zero, so we
want all of the positive
sections. Notice that x ≠ 1 and x ≠ 4
because it would make the original
problem undefined, so you should use
an open circle at x = 1 and
x = 4 instead of a closed circle to draw
the graph.

Step 6: Use interval notation to


write the final answer.

34
Example 3: Graph:

Step 1: Write the inequality in


the correct form. One side must
be zero and the other side can
have only one fraction, so simplify
the fractions if there is more than
one fraction.
Step 2: Find the key or critical
values. To find the key/critical
values, set the numerator and
denominator of the fraction equal
to zero and solve.
Step 3: Make a sign analysis
chart. To make a sign analysis
chart, use the key/critical values
found in Step 2 to divide the
number line into sections.
Step 4: Perform the sign analy-
sis. To do the sign analysis, pick
one number from each of the sec-
tions created in Step 3 and plug
that number into the polynomial to
determine the sign of the resulting
answer. In this case, you can
choose x = –6 which results in
–0.031, x = –3 which results in
+0.4, x = 0 which results in –1.25,
and x = 3 which results in +1.6.
Step 5: Use the sign analysis
chart to determine which sections
satisfy the inequality. In this case,
we have less than zero, so we
want all of the negative sections.
Step 6: Use interval notation to
write the final answer.

35
Application
A. Determine whether the following expression is a rational function, a rational
equation, a rational inequality or none of these.

1. ________________________
2. ________________________
3. ________________________

4. ________________________
5. ________________________

B. Solve each equation and inequality.


1. Solve:

2. Solve:

3. Solve:

4. Solve:

5. Solve:

Reflection
This lesson teaches you to differentiate rational functions, rational equations
and rational inequalities. This lesson also allows you to solve rational equations
and inequalities.
Through this lesson, what insights have you gained regarding the importance
of following a step-by-step procedure in doing a certain task in order to succeed?
--------------------------------------------------------------------------------------------------

36
Posttest
Read and analyze each of the following then shade the circle that
corresponds to the letter of the best answer.

A B C D
O O O O 1. Which of the following is a rational function?

A. C.

B. D.

O O O O 2. Which of the following is a rational equation?

A. C.

B. D.

O O O O 3. Which of the following is a rational inequality?

A. C.

B. D.

O O O O 4. Solve for x:

A. x = 6/7, x = -3 C. x = -6/7, x = 3

B. x = -6/7, x = -3 D. x = 6/7, x = 3

O O O O 5. Solve for x:

A. C.

B. D.

----------------------------------------------------------------------------------------------------
End of Lesson 4

37
Lesson 5: Rational Functions
----------------------------------------------------------------------------------------------------
Pretest
Read and analyze each of the following then shade the circle that
corresponds to the letter of the best answer.
A B C D
O O O O 1. Which of the following is a polynomial function?
A. C.
B. D.
O O O O 2. Which of the following is the correct table of values of the rational
function ?

x -3 -2 -1 0 1 2 3
A. y 0.75 0.67 0.5 0 und 2 1.5

x -3 -2 -1 0 1 2 3
B. y 0.75 0.67 0.5 und 0.5 -2 -1.5

x -3 -2 -1 0 1 2 3
C. y -0.75 -0.67 -0.5 0 und 2 1.5

x -3 -2 -1 0 1 2 3
D. y 0.75 0.67 -0.5 und -0.5 -2 -1.5

O O O O 3. Which of the following is the graph of ?

A. B.

38
C. D.

O O O O 4. Which of the following is the correct table of values of the rational


function ?
x -3 -2 -1 0 1 2 3
A.
y 0.71 0.8 1 2 -1 0.71 0.8

x -3 -2 -1 0 1 2 3
B.
y 0.71 0.8 1 2 -1 0 0.2

x -3 -2 -1 0 1 2 3
C. y 0.71 0.8 -1 -2 -1 0 0.2

x -3 -2 -1 0 1 2 3
D. y -0.71 -0.8 1 2 und 0 0.2

O O O O 5. Which of the following is the graph of ?


A. B.

39
C. D.

------------------------------------------------------------------------------------------------------------
Competencies
1. Represents real-life situations using rational functions. M11GM-Ib-1
2. Represents a rational function through its: (a) table of values, (b) graph,
and (c) equation. M11GM-Ib-4

Objectives
At the end of the lesson, the learners are expected to:
1. Show illustrations of rational functions to represent real-life situations.
2. Construct table of values to represent rational function.
3. Show the graph of rational functions.
4. Write equations of rational functions.

Procedure/Learning Experience
Activity
A. Identify the polynomial functions from the given set of expressions.
1.

2.

3.

5.

6.
B. Given the polynomial function , find
1. The degree of the polynomial
2. The leading coefficient
3. The constant term

40
Analysis
1. What have you realized while doing the activity?
2. How do we identify polynomial functions?
3. How will you differentiate a polynomial function from a rational function?
Are they related? Why?

Abstraction
A polynomial function p of degrees n is a function that can be written in
the form: . Where a 0, a1…an є Ɽ, an ≠ 0,
and n is a positive integer. Each addend of the sum is a term of the polynomial
function. The constants a0, a1, a2,…an are the coefficients. The leading coefficient
is an. The leading term is , and the constant term is a0.
Rational Function
A rational function r(x), is a function in the form of ,
where p(x) and q(x) are polynomial functions, and q(x) ≠ 0.

In dealing with a rational function, it is often helpful to identify values for which
the rational function is undefined. Later, one will find that the graph of the rational
function approaches lines that contain these values for which the rational function
is undefined; hence the graph does not cross these lines.
Example 1: Construct a table of values and sketch the graph of the
following rational functions:

a.

b.

c.

41
Solution:
a.

x -5 -4 -3 -2 -1 0 1 2 3 4 5

y -0.20 -0.25 -0.33 -0.5 -1 undefined 1 0.5 0.33 0.25 0.20

Graph:

b.

x -5 -4 -3 -2 -1 0 1 2 3 4 5

y -0.14 -0.17 -0.20 -0.25 -0.33 -0.50 -1 und 1 0.50 0.33

Graph:

42
c.

x -5 -4 -3 -2 -1 0 1 2 3 4 5
y 1.25 1.33 1.50 2 und 0 0.50 0.67 0.75 0.80 0.83

Graph:

Example 2 : An object is to travel a distance of 20 meters. Express velocity


v as a function of travel of time t in seconds.
Solution:
The following table of values shows v for various values of t.

t (seconds) 1 2 4 5 10
V (meter per second) 20 10 5 4 2

The function can be represent v as a function of t.

Example 3 : Suppose that (in mg/mL) represents the


concentration of a drug in a patient’s bloodstream t hours after the drug was
administered. Construct a table of values for c(t) for t = 1,3,7, and 10. Round off
answers to three decimal places. Use the table to sketch a graph and interpret the
results.
t 1 3 7 10
Solution: c(t) 2.5 1.5 0.7 0.495
The following table of values shows c for various values of t.

43
Graph

The graph indicates that the maximum drug concentration occurs around 1
hour after the drug was administered. After 1 hour, the graph suggests that drug
concentration decreases until it is almost zero.

Application
A. Solve the following.
1. Given
a. Construct a table of values using the numbers from -2 to 8.
b. Plot the points in the Cartesian plane and determine whether the
points form a smooth curve or a straight line.

2. Represent the speed of a runner as a function of the time t it takes to run


200 meters in the track. Construct a table of values for the speed of a
runner and sketch a graph by plotting and connecting the points.

3. A hypothetical function c(t) representing the concentration of a drug in a


patient’s bloodstream over time t (in hours) is given as .
a. Construct a table of values.
b. Plot the points in a Cartesian plane and connect them.
c. What can you say about the function?

Reflection
This lesson teaches you to the real-life application of rational functions and
how it could be represented through tables, graphs and equations.
In this topic, you have realized that it is not that hard to solve problems
involving functions. Give your insights.
------------------------------------------------------------------------------------------------------------

44
Posttest
Read and analyze each of the following questions then shade the circle that
corresponds to the letter of the best answer.
A B C D
O O O O 1. Which of the following is a polynomial function?
A. C.

B. D.

O O O O 2. Which of the following is the correct table of values of the


rational function ?
x -3 -2 -1 0 1 2 3
A.
y 0.75 0.67 0.5 0 und 2 1.5

x -3 -2 -1 0 1 2 3
B. y 0.75 0.67 0.5 und 0.5 -2 -1.5

x -3 -2 -1 0 1 2 3
C. y -0.75 -0.67 -0.5 0 und 2 1.5

x -3 -2 -1 0 1 2 3
D. y 0.75 0.67 -0.5 und -0.5 -2 -1.5

O O O O 3. Which of the following is the graph of ?


A.

45
B.

C.

D.

46
O O O O 4. Which of the following is the correct table of values of the rational
function ?

x -3 -2 -1 0 1 2 3
A.
y 0.71 0.8 1 2 -1 0.71 0.8

B. x -3 -2 -1 0 1 2 3
y 0.71 0.8 1 2 -1 0 0.2

x -3 -2 -1 0 1 2 3
C.
y 0.71 0.8 -1 -2 -1 0 0.2

x -3 -2 -1 0 1 2 3
D.
y -0.71 -0.8 1 2 und 0 0.2

O O O O 5. Which of the following could be the graph of ?

A.

B.

47
C.

D.

------------------------------------------------------------------------------------------------------------
End of Lesson 5

48
Lesson 6: Representations of Rational Function
--------------------------------------------------------------------------------------------------
Pretest
Shade the circle corresponding to the letter of the best answer.
A B C D
O O O O 1. It is a relation from a set of inputs to a set of possible outputs
where each input is related to exactly one output.
A. Function C. Set
B. Relation D. Subset

O O O O 2. It is a family of functions where y = belongs.


A. Trigonometric C. Exponential
B. Logarithmic D. Rational

O O O O 3. It is a graphic organizer or chart that helps you determine two or


more points that can be used to create your graph.
A. Graphic Organizer C. Table of Values
B. Equation D. Graph

O O O O 4. It is a collection of points and lines connecting some (possibly


empty) subset of them.
A. Graphic Organizer C. Table of Values
B. Equation D. Graph

O O O O 5. It is a statement that the values of two mathematical expressions


are equal.
A. Graphic Organizer C. Table of Values
B. Equation D. Graph
----------------------------------------------------------------------------------------------------------

49
Competency
Represents a rational function through its: (a) table of values,
(b) graph, and (c) equation. M11GM-Ib-4

Objective
At the end of the lesson, the learners are expected to:
1. Construct table of values and graph of a rational function.

Procedure/Learning Experience
Activity
Read the story then answer the following questions below.

The Maya Birds and the Pitcher (Fill me in!)

Once upon a time, there were thirsty Maya birds. They came upon a red
pitcher that had some water in it, but when they put their beak into the pitcher, they
found they could not reach the water. Then, one of the Maya birds had an idea.
She looked around, found a pebble and dropped it into the pitcher. Water level rose
a little bit. The other birds were encouraged and joined her to continue to drop
pebbles into the pitcher, one at a time, until the water rose up high enough for them
to reach it with her beak. The birds drank and were satisfied.
In this selection, you will simulate the story of “The Maya Birds and the
Pitcher” using a graduated cylinder and marbles.
Fill your graduated cylinder with 80mL of water. You will be adding marbles
until the water reaches a level of at least 100mL – the level at which the Maya birds
can reach the water.
Add the marbles one at a
time and NUMBER OF MARBLES WATER LEVEL (mL) record the effect
that each (x) (y) additional
marble has 0 on the water
1
level. Complete the
2
table.
3
4
5

50
Abstraction
Example 1: Represent the speed of a runner as a function of the time it
takes to
run 100 meters in track.
Solution:
Since the speed of a runner depends on the time it takes the runner to run
100 meters, we represent speed as a function of time. Let x represent the time it
takes the runner to run 100 meters. Then the speed can be represented as a
function s(x) as follows:
Observe that similar structure to the formula relating speed, distance
and time.

Example 2: Continuing the scenario above, construct a table of values for


the
speed of a runner against different run times.
Solution:
A table of values can help us determine or identify the behavior of a function
as the variable x changes.
The current world record (As of October 2015) for the 100meter dash is 9.58
seconds set by the Jamaican Usain Bolt in 2009. We start our table of values at
10 seconds. Let x be the run time and s(x) be the speed of the runner in meters
per second, where . The table of values for run time from 10 seconds
to 20 seconds is as follows.
From xthe table,10
we can observe
12 that
14the speed16 decreases
18 with time.
20 We can
use a graph to determine if the points on the function follow a smooth curve or a
s(x) 10 8.33 7.14 6.25 5.56 5
straight line.

Example 3. Plot the points of the obtained table of values on a Cartesian


Plane. Determine if the points on the function follow a smooth curve
or a straight line.

Solution:
Assign points on the Cartesian Plane for each entry on the table of values
above. A(10,10), B(12,8.33), C(14, 7.14), D (16, 6.25), E(18, 5.56) and F(20,5)
Plot these on a Cartesian Plane.

51
By connecting the points, we can see that they are not collinear but rather follow a
smooth curve.

For the 100 meter scenario, we have constructed a function of speed against
time, and represented our function with table of values and a graph.

The example is based on a real world scenario and has limitations on the
values of x-variables. For example, a runner cannot have negative time
(which would mean he is running backwards in time), nor can exceed the limits of
human physiology (a person can run 100meters in 5 seconds). However, we can
apply the skills in constructing the table of values and graphs to observe the
behavior of rational functions.

Application
Represent the rational function given by using the table of values
and plot a graph of the function by connecting points.

Reflection
Functions are everywhere and it can be applied through technology. For
example:
1. Mapping images to the names of the people on the images
2. Mapping digital songs to authors/genre/song name
3. Mapping internet clicks to advert suggestion
4. Mapping Google answers to subjects/categories. Can you cite other
examples showing real-life applications of functions?
————————————————————————————————————

52
Procedure/Learning Experience
Activity
What am I?
Plot the following points on the grid. Then draw a straight line to connect from one
to next.
1. (8,6) to (4,10) 6. (8,6) to (4,6)
2. (4,2) to (8,2) 7. (4,2) to (2,4)
3. (4,4) to (10,4) 8. (8,2) to (10,4)
4. (2,4) to (4,4) 9. (4,4) to (4,6)
5. (4,6) to (4,10)

Analysis
1. What image was formed in the graph?
2. How did you locate the points?
3. What did you consider in plotting the points?

Abstraction
Consider the function f(x) = . Find it’s domain, range, and intercepts.
Sketch the graph.

Solution:
A. The domain of f(x) is {x Є R | x ≠ -2}.
The function is undefined at x=-2. This means that x=-2 is not a part of

55
Posttest
Shade the circle corresponding to the letter of the best answer.
A B C D
O O O O 1. It is in the form of f(x) = where p(x) and q(x) are polynomial
functions and q(x) is not equal to zero.
A. Rational Equality C. Rational Function
B. Rational Inequality D. None of these

O O O O 2. Complete the table using the equation f(x) =


A. 12 C. 16
B. 14 D. 18

O O O O 3. Martha has won 19 out of 28 tennis matches this season. Which


equation models suggest how many more games she needs to
win to average 75% wins over
x 1 2 3 4
loses?
f(x) 1 4 9 ?

A. C.

B. D.

O O O O 4. In order to get a college tennis scholarship, Martha needs to


bring her winning average to 80%. What is the number of match
es she needs to win given that she already won 19 out of 28?
A. 3 C. 17
B. 4 D. 22

O O O O 5. Joel is working on his chemistry project and he has 300mL of


12% acid solution. If he needed to decrease the acidity of the
solution, which of the following is the correct function that would
show the new acidity of the solution given x mL of water added?

A. C.

B. D.
-------------------------------------------------------------------------------------------------------------

53
Lesson 7: Graphing Rational Function
----------------------------------------------------------------------------------------------------
Pretest
Shade the circle corresponding to the letter of the best answer.
A B C D
O O O O 1. It is the set of all values that the variable x in a function can take.
A. Domain C. Zeroes
B. Range D. Intercepts

O O O O 2. It is the values of x which make the function zero.


A. Domain C. Zeroes
B. Range D. Intercepts

O O O O 3. It is the set of values that f(x) can take.


A. Domain C. Zeroes
B. Range D. Intercepts

O O O O 4. It is the function value when x = 0.


A. x-intercept C. Horizontal Asymptotes
B. y-intercept D. Vertical Asymptotes

O O O O 5. It is a diagram showing the relation concerning the variables


measured along one pair of axes at right angles. It is a plot or
trace on a Cartesian Plane.
A. Graph C. Table of Values
B. Points D. None of the above.
------------------------------------------------------------------------------------------------------------
Competencies
1. Finds the domain and range of a rational function. M11GM-Ib-5
2. Determines the: (a) intercepts (b) zeroes; and (c) asymptotes of
rational functions. M11GM-Ic-1
3. Graphs rational functions. M11GM-Ic-2
4. Solves problems involving rational functions, equations, and
inequalities. M11GM-Ic-3

Objectives
At the end of the lesson, the learners are expected to:
1. Identify the domain and range of a rational function.

54
B. The x-intercept of f(x) is 2 and its y-intercept is –1.
Recall that the x-intercepts of a rational function are the values of x that
will make the function zero. A rational function will be equal zero if its
numerator is zero. Therefore, the zeroes of a rational function are the
zeroes of its numerator. The numerator x-2 will be zero at x = 2.
Therefore, x=2 is a zero of f(x). The y-intercept of a function is equal to
f(0) = .

C. To sketch the graph of f(x), let us look at what happens to the graph
near the values of x which makes the denominator undefined.
Definition. VERTICAL ASYMPTOTE
The vertical line x=a is a vertical asymptote of a function f if the graph
increases or decreases without bound as the x values approach a from the
right or left.

How to Solve for the Vertical Asymptote?


1. Reduce the rational function to lowest term by cancelling out the common
factors in the numerator and the denominator.
2. Find the values a that will make the denominator of the reduced rational
function equal to zero.
3. The line x = a is a vertical asymptote.

Since f(x)= is already in simplest term, let us take at the value


that will make the denominator zero, If x + 2 = 0. Therefore x = -2 is the vertical
asymptote.
Definition. HORIZONTAL ASYMPTOTE
The horizontal line y=b is a horizontal asymptote of the function f if f(x)
gets closer to b as x increases or decreases without bound.
How to find the horizontal asymptote of a rational function?

Given that n and m are the degrees of the numerator and of the denominator,
respectively:
 If n < m, the horizontal asymptote is y = 0
 If n = m, the horizontal asymptote is y = , where a is the leading
coefficient of the numerator, and b is the leading coefficient of the
denominator.

56
Note: A rational function may or may not cross its horizontal asymptote. If the
function does not cross the horizontal asymptote y = b, then b is not part of the
range of the rational function.

In this case, the degree of the numerator and the denominator is equal, therefore,
we will apply the second case. That is, y = . From the example f(x) = , the
horizontal asymptote is y = = 1.
Construct a table of signs to determine the sign of the given function on the
intervals determined by the zeroes and the vertical asymptotes. That is, construct
a table of values of x that will make either the numerator or denominator 0 as
boundaries. In this example, the boundaries are x = -2 and x = 2.

Plot the zeroes, the intercepts, and the asymptotes.

Assign other
points for precise
tracing of the curves.
Do not cross the
vertical and horizontal
asymptotes.

57
Application
Using the rational function , identify the following.
Intercepts: _______________________________________________
Asymptotes: ______________________________________________
Domain and range: _________________________________________
Graph

Reflection
An application of rational functions may involve the number of persons who
can do a task in a certain amount of time. We can handle these applications
involving work in a manner similar to the method we used to solve distance,
speed, and time problems. Work = Rate x Time. Suppose you can finish a report
in 2 hours. Your classmate can finish the same report in 4 hours. How long will it
take to finish the report if both of you work together? We have a saying that “Two
heads are better than one”, would you rather work alone or with a team? Why?
------------------------------------------------------------------------------------------------------------

58
Posttest
Shade the circle corresponding to the letter of the best answer.
A B C D
O O O O 1. What is the vertical asymptote of the equation y=
A. x = 6 C. x = 2
B. x = 7 D. x = -2

O O O O 2. What is the horizontal asymptote of the function


A. y = 0 C. y = 1
B. y = 4 D. None

O O O O 3. What is the domain of the given rational expression,


A. {x | x ϵ R, x ≠ -2} C. {x | x ϵ R, x ≠ -4}
B. {x | x ϵ R, x ≠ 2} D. {x | x ϵ R, x ≠ 4}

O O O O 4. What is a line that approaches the graph continually but never


touches it?
A. x intercept C. asymptote
B. y intercept D. inverse function

O O O O 5. Which of the following graphs represents a function?

A. C.

B. D.

------------------------------------------------------------------------------------------------------------

59
“One child, one teacher, one book, one pen
can change the world”
——————- Malala Yousafzai ———————

LESSON 8: One-to-one Functions


LESSON 9: Representations of Inverse Functions
LESSON 10: Domain and Range of Inverse Functions
LESSON 11: Graphs of Inverse Functions
LESSON 12: Problems Involving Inverse Functions
CHAPTER III - ONE-TO-ONE FUNCTIONS
Lesson 8: One-to-One Functions
----------------------------------------------------------------------------------------------------
Introduction
In this topic, the learners are expected to demonstrate understanding of key
concepts of inverse functions, exponential functions, and logarithmic functions.
Learners should also be able to apply concepts of inverse functions, exponential
functions, and logarithmic functions to formulate and solve real-life problems with
precision and accuracy.

Directions for the User


A pretest is prepared for you to assess which part of the lesson you need to
give more focus on. After answering it, you may then proceed to the Self-Learning
Module proper consisting of the Procedure/Learning Experience and Reflection.
Before leaving this module, you will also answer a posttest. Ready? Begin!
------------------------------------------------------------------------------------------------------------
Pretest
Shade the circle corresponding to the letter of the best answer.
A B C D
O O O O 1. It is a rule which associates each element of set A with at least
one element in set B.
A. Function C. Set
B. Relation D. Subset
O O O O 2. It is a rule which uniquely associates elements of one
set A with the elements of another set B; each element in
set A maps to a single element in set B.
A. Function C. Set
B. Relation D. Subset
O O O O 3. It associates two or more values of the independent (input)
variable with a single value of the dependent (output) variable.
A. One-to-one C. Many-to-one
B. One-to-many D. Many-to-many
O O O O 4. It is a single x-value that relates to two different y-values.
A. One-to-one C. Many-to-one
B. One-to-many D. Many-to-many
O O O O 5. A single x-value relates to only one unique y-values.
A. One-to-one C. Many-to-one
B. One-to-many D. Many-to-many

61
Competency
Represents real-life situations using one-to-one functions. M11GM-Id-1

Objectives
At the end of the lesson, the learners are expected to:
1. Verify a one-to-one function.
2. Determine if a function is a one-to-one.

Procedure/Learning Experience
Activity
Ask five (5) learners to write their LRN on the table provided below.
Name of the Member LRN

Analysis
1. What did you observe from the table? Did you notice any repeated LRN?
2. What do you think is the reason why learners have their own LRNs?
3. What kind of function is depicted from the given activity?

Abstraction
One-to-One Functions
A function f is one-to-one if it never takes the same value twice or
. That is, the same y-value is never paired
with two different x-values.
In the Venn diagram below, function f is a one-to-one since not two inputs
have a common output.

Figure 1. Venn diagram of a one-to-one function.

62
In the Venn diagram below, function f is NOT a one-to-one since the inputs -
1 and 0 have the same output.

Figure 2. Venn diagram of a function that is not a one-to-one.


On the other hand, the function g(x) = is not a one-to-one function,
because g(−1) = g(1). There are a lot of real-life applications of a one-to-one
function. Determine whether the given relation is a function. If it is a function,
determine whether it is one-to-one.

Example 1: The relation pairing an SSS member to his or her SSS number.
Solution:
Each SSS member is assigned a unique SSS number. Thus, this relation is
a function. Further, two members cannot be assigned the same SSS number,
therefore, the function is one-to-one.

Example 2: The relation pairing a citizenship to a person.


Solution:
The relation is a function because each person has a citizenship. However,
a person can have two citizenship, (dual citizen) therefore, it is not one-to-one
function.

Graph of a One-to-one Function


If f is a one-to-one function then no two points , have the
same y-value. Therefore, no horizontal line cuts the graph of the equation y = f(x)
more than once.
Example. Compare
the graphs of the
above functions

63
How to Determine if a Function is One-to-One
Horizontal Line test: A graph passes the Horizontal Line Test if each
horizontal line cuts the graph at most once.
A function f is one-to-one if and only if the graph y = f(x) passes the
Horizontal Line Test.
Example. Which of the following functions are one-to-one?

Application
Determine whether each of the following situations is a one-to-one function.
Elaborate your answer.
1. The relation of a dog to its family members.
____________________________________________________________
__________________________________________________________
___________________________________________________________

2. The relation of a person to his or her passport.


____________________________________________________________
__________________________________________________________
___________________________________________________________

3. A car model to its manufacturer company.


____________________________________________________________
__________________________________________________________
___________________________________________________________

Reflection
This lesson helps you understand the applications of one-to-one function in
real-life situations. You also have learned how to determine whether a graph is a
one-to-one function or not by using the Horizontal Line Test. In this connection,
suppose you see two people who are in a relationship, and you know that one of
the two has another relationship, do you think what they have is a one-to-one
function? Why?
------------------------------------------------------------------------------------------------------------

64
Posttest
Shade the circle corresponding to the letter of the best answer.
A B C D
O O O O 1. Which of the following is not a one-to-one function?

A.

B.

C.

D.

O O O O 2. Which type of relation wherein every element in the domain is


paired with exactly one element in the range?
A. Function C. Inverse
B. Asymptote D. Composite

O O O O 3. Which of the following relationships DOES NOT indicate a


one-to-one function?
A. A tricycle and its plate number
B. Chemical symbol to its chemical element
C. Parents and their children
D. Husband and Wife

O O O O 4. Which of the following represents a one-to-one function?


A. Teacher to students
B. Student to their LRN
C. Mother to her children
D. Students to teacher

65
O O O O 5 Which of the following graphs represents a one-to-one function?
A.

B.

C.

D.

------------------------------------------------------------------------------------------------------------
End of Lesson 8

66
Lesson 9: Representations of Inverse Function
------------------------------------------------------------------------------------------------------------
Pretest
Shade the circle corresponding to the letter of the best answer.
A B C D
O O O O 1. It is a test used to determine whether the function is one-to-one
or not.
A. Horizontal Line Test C. T-test
B. Vertical Line Test D. z-test

O O O O 2. It is a type of function wherein no points on the x and y values


are repeated.
A. One-to-one C. Many-to-one
B. One-to-many D. Many-to-many

O O O O 3. Identify the opposite of the word FAST.


A. Rapid C. Slow
B. Hurry D. Rush

O O O O 4. Which of the following graphs is not a one-to- one func-


tion?
A. C.

B. D.

O O O O 5. Which of the following pairs is not inverses of each other?


A. 1 and -1 C. and 2
B. 0 and -0 D. 3 and
------------------------------------------------------------------------------------------------------------

67
Competencies:
1. Determines the inverse of a one-to-one function. M11GM-Id-2
2. Represents an inverse function through its: (a) table of values, and
(b) graph. M11GM-Id-3

Objectives:
At the end of the lesson, the learners are expected to:
1. Verify an inverse of a one-to-one function.
2. Construct table of values and graph of an inverse function.

Procedure/Learning Experience
Activity
In the Philippine setup, to determine the temperature in degrees we use
Celsius( ). However, in the United States of America, they use the Fahrenheit
( Imagine you are in this country, when given the temperature in degrees
Celsius, for example 12 , and you were asked by your co-worker to convert it in
Fahrenheit using this formula, = + 32. What will you do?

Analysis
1. What happens if you want to input Fahrenheit and output degree Celcius?
2. From = + 32, what will be the formula in computing for C?

Abstraction
Two functions are said to be inverses of each other if they are the reverse
process of each other. The function that reverses or interchanges the roles of x and
y is the inverse function of f and is denoted . .
More formally, if a function = ( ) assigns values of the input quantity x to
values of the output quantity y, the inverse of f, denoted is the function
which assigns output values of f to corresponding input values.

Figure 2http://rpdp.net/admin/images/uploads/resource_10551.pdf

68
How to determine the inverse of a function?
1. Determine if the function is one-to-one. (NOTE: a function has an inverse if
and only if it is a one-to-one function)
2. Interchanging the x and y variables. This new function is the inverse of the
function.
3. If the result is an equation, solve the equation for y in terms of x. Replace y
with , symbolizing the inverse function or the inverse of f.

Example 1: Find the inverse of .


Solution:
The equation of a function is y=3x+1. Interchanging the x and y variables, we get
x = 3y+1. Solving y for x:

Therefore, the inverse of is

Example 2: x -3 -2 -1 0 1 2 3 Find the inverse of the


table provided y 9 2 6 -4 -5 -8 -9 below.

Solution: We can observe that the


table is a one- x 9 2 6 -4 -5 -8 -9 to-one function since
no values of x y -3 -2 -1 0 1 2 3 and y has been
repeated. We can now find its
inverse. To find the inverse, switch the values of x to y and y to x.

Example 3: Find the inverse of the function


Solution: First we write n in equation form: . We can then reverse the
roles of x and y, we get .

The equation represents the inverse of the function and it can

69
In figure below both and are graphed, along with the line y = x.
As you can see, the graph of is the graph of reflected about the
line y = x.

In this figure, we can see that the line of y = x is graphed in red while the
function is graphed in blue while its inverse which is is
graphed in green. Notice that the graph of the inverse of a function is the same as
the graph of the original function but reflected about the line y = x .

Application
Determine whether the two functions are inverses of each other. Use the first
function to prove that the second function is its inverse.
1. and

2. and

Reflection
This lesson helps you understand the applications of inverse function. You
have understood that inverse function is a function that reverses another function.
In life, if it so happens that you have done some mistakes, you can only correct it
and not reverse it. But if you would be given a chance to reverse one thing in your

70
Posttest
Shade the circle corresponding to the letter of the best answer.
A B C D
O O O O 1. Which of the following is the inverse
of the table x -5 -3 0 3 below?
y 1 2 3 4
x -5 -3 0 3 x -5 -3 0 3
y 1 2 3 4 y -1 -2 -3 -4
A. C.
x 1 2 3 4 x 1 3 2 4
B. y -5 -3 0 3 D. y -5 -3 0 3

O O O O 2. What is the opposite or the reverse of a function?


A. Function C. Inverse
B. Asymptote D. Composite
O O O O 3. What is the inverse of the function
A. C.
B. D.
O O O
O 4. Which of the following scenarios DOES NOT depict an inverse
function?
A. Tying and untying shoe laces
B. Wrapping and unwrapping of a present
C. Breaking and fixing of a glass
D. Buttoning and unbuttoning of a dress
O O O O 5. Which of the following graphs represents the inverse of

A. C.

B. D.

71
Lesson 10: Domain and Range of Inverse Function
------------------------------------------------------------------------------------------------------------
Pretest
Shade the circle corresponding to the letter of the best answer.
A B C D
O O O O 1. What is the inverse one-to-one function of f(x) = {(1,9), (2,8),
(3,7), (4,6)}?
A. f-1(x) = {(-1,-9), (-2,-8), (-3,-7), (-4,-6)}
B. f-1(x) = {(9,1), (8,2), (7,3), (6,4)}
C. f-1(x) = {(-9,-1), (-8,-2), (-7,-3), (-6,-4)}
D. f-1(x) = {(-1,9), (-2,8), (-3,7), (-4,6)}

O O O O 2. What is the domain of the function f(x) = {(3,9), (-6,8), (9,7),


(-12,6), (15,5)}?
A. {3, -6, 9, -12, 15} C. {9, 8, 7, 6, 5}
B. {3, 6, 9, 12, 15} D. {all real numbers}

O O O O 3. What is the range of the function f(x) = {(3,9), (-6,8), (9,7), (-12,6),
(15,5)}?
A. {3, -6, 9, -12, 15} C. {5, 6, 7, 8, 9}
B. {3, 6, 9, 12, 15} D. {all real numbers}

O O O O 4. What is the domain of the function f - 1 (x) = x - 5?


A. {X Є ℝ| -5 ≤ x ≤ 5} C. {-4, -1, 4, 11, 20, 31}
B. {X Є ℝ| -5 ≤ y ≤ 5} D. {X Є ℝ}

O O O O 5. What is the range the function f-1(x) = 3x + 2?


A. {-4, -1, 2, 5, 8, 11} C. {X Є ℝ | except - 1/3}
B. {X Є ℝ | -2 ≤ y ≤ 3} D. {X Є ℝ}
------------------------------------------------------------------------------------------------------------

72
Competency
Finds the domain, and range of an inverse function. M11GM-Id-4

Objectives
At the end of the lesson, the learners are expected to:
1. Determine the domain and range of inverse functions.
2. Identify real-life examples of domain and range.

Procedure/Learning Experience
Activity
Investigate and determine if the following data show one-to-one function or not.
Write O if it is One-to-one and N if Not. Justify your answer. Use the space provided
before each item.
________1. People to their birthdays
________2. People to their Social Security System number
________3. People to their place of residence
________4. Vehicles to plate numbers
________5. Movie tickets to seat numbers

Analysis
1. What have you noticed with every pair of variables?
2. Which is dependent and independent variable?
3. How do you find the domain and range?

Abstraction
Inverse function
1. If f(x) is a one-to-one function with ordered pairs of the form (x,y), its
inverse function, f -1(x), is a one-to-one function with ordered pairs of the
form (y,x).
2. To determine the inverse of a linear function, you need to derive the function
that “undoes” the original function.
3. If your function is defined as a list of ordered pairs, simply swap the x and y
values. Remember, the inverse will be a function if the original function is
one- to-one function.

73
Examples:
1. Given function f, find the inverse. Is the inverse also a function? f(x) = {(3,4),
(4,5), (5,6), (6,7)}.
Answer: Function f is a one-to-one function, since the x and y values are
used only once. The inverse is f -1(x) = {(4,3), (5,4), (6,5), (7,6)}. The domain is
the values in x and the range in y. The domain of the function becomes the
range of the inverse function and the range of the function is the domain of the
inverse. The domain of the inverse one-to-one function is {4,5,6,7} and the range
is {3,4,5,6}.

2. Given function f, f -1(x) = x + 7, what is the domain and range of the given
inverse function?
To determine the domain and range of an inverse function, you may use the
table of values
x -2 -1 0 1 2 3 4
below - -1 and complete
f (x)
the values:

Solution:
If x = -2, If x = -1, If x = 0 If x = 1
f- -1(x) = x + 7 f-1(x) = x + 7 f- -1(x) = x + 7 f-1(x) = x + 7
f- -1(x) = (-2) + 7 f -1(x) = (-1) + 7 f- -1(x) = (0) + 7 f-1(x) = (1) + 7
f- -1(x) = 5 f- -1(x) = 6 f- -1(x) = 7 f-1(x) = 8

If x = 2 If x = 3 If x = 4
f-1(x) = x + 7 f-1(x) = x + 7 f-1(x) = x + 7
f-1(x) = (2) + 7 f-1(x) = (3) + 7 f-1(x) = (4) + 7
f-1(x) = 9 f-1(x) = 10 f-1(x) = 11
x -2 -1 0 1 2 3 4
f- -1(x) 5 6 7 8 9 10 11 Answer:
The domain is {X Є
ℝ}, and the range is {all real numbers}.

3. What is the domain and the range of the inverse function, f-1(x) = ?

74
x -3 -2 -1 0 1 2 3
- -1
f (x)

If x = -3 f-1(x) = x - 3 = (-3) - 3 = -6 = -2
3 3 3
-1
If x = -2 f (x) = x - 3 = (-2) - 3 = -5
3 3 3
-1
If x = -1 f (x) = x - 3 = (-1) - 3 = -4
3 3 3
-1
If x = 0 f (x) = x - 3 = (0) - 3 = -3 = -1
3 3 3
-1
If x = 1 f (x) = x - 3 = (1) - 3 = 1 - 3 = -2
3 3 3 3
-1
If x = 2 f (x) = x - 3 = (2) - 3 = 2-3 = -1
3 3 3 3
If x = 3 f -1(x) = x - 3 = (3) - 3 = 0
3 3

Answer:
The domain of the f -1(x) = x - 3 is {X Є ℝ}, and the range is {Y Є ℝ}.

x -3 -2 -1 0 1 2 3
-1
f (x) -2 -5/3 -4/3 -1 -2/3 -1/3 0
3

Application
Determine the domain and range of the following inverse functions.
1. f -1(x) = x + 1

2. f -1(x) = 2x - 7

3. f -1(x) = 3x - 3

4. f -1(x) = 5x - 1

75
Reflection
Remember that an inverse function is a one-to-one function. Whereas, the
domain of the inverse function is the range of the one-to-one function and the
range of the inverse function is the domain of the one-to-one function.
-------------------------------------------------------------------------------------------------------------
Posttest
Shade the circle corresponding to the letter of the best answer.
A B C D
O O O O 1. What is the inverse one-to-one function of f(x) = {(-3,-3), (-2,-4),
(-1,-5), (0,-6), (1,-7)}?
A. f-1(x) = {(-3,-3), (-4,-2), (-5,-1), (0,-6), (-7,1)}
B. f-1(x) = {(-3,-3), (-4,-2), (-5,-1), (-6,0), (-7,1)}
C. f-1(x) = {(-3,-3), (-2,-4), (-1,-5), (-6,0), (1,-7)}
D. f-1(x) = {(-3,-3), (--2,-4), (-5,-1), (-6,0, (1,-7)}

O O O O 2. What is the domain of the f -1(x) = {(-1,-9), (0,-8), (1,-7), (2,-6)}?


A. {-1, 0, 1, 2} C. {X Є ℝ}
B. {-9, -8, -7, -6} D. none of these

O O O O 3. What is the range of the f-1(x) = {(-2,2),(-1,5), (0,8), (1,11), (2,14)}?


A. {-2, -1, 0, 1, 2} C. {Y Є ℝ}
B. {2, 5, 8, 11, 14} D. none of these

O O O O 4. What is the domain of the f-1(x) = 9x + 5?


A. {X Є ℝ| -5 ≤ x ≤ 5} C. {-4, -1, 4, 11, 20, 31}
B. {X Є ℝ| -5 ≤ y ≤ 5} D. {X Є ℝ}

O O O O x -2 -1 0 1 2 3 5. What
-1
f (x) is the
range
the function f -1(x) = 5x + 2?

A. A = {-8, -3, 2, 7, 12, 17} C. {X Є ℝ | except 2/3}


B. {X Є ℝ | -2 ≤ y ≤ 3} D. {Y Є ℝ}
-------------------------------------------------------------------------------------------------------------
End of Lesson 10

76
Lesson 11: Graphs of Inverse Function
-------------------------------------------------------------------------------------------------------------
Pretest
Shade the circle corresponding to the letter of the best answer.
A B C D
O O O O 1. Which of the following doesn’t belong to the group?
A. C.

B. D.

O O O O 2. Which of the following is best described on the graph of the


one-to-one function and its inverse function?
A. Hyperbolic C. Parallel
B. Parabola D. Symmetric

O O O O 3. This equation is used to test if the graph of one-to-one function is


its inverse function.
A. f(x) = y C. f(x) = x
B. f(x) = x - y D. f(x) = x + y

77
O O O O 4. Which of the following ordered pairs of inverse function has its
one-to-one function indicated in the table below:
f (x) 3 4 5 6 7
y -2 -1 0 1 2

A. f - 1(x) = {(3,-2), (4,-1), (5,0), (6,1), (7,2)}


B. f - 1(x) = {-2,3), (-1,4), (0,5), (1,6), (2,7)}
C. f - 1(x) = {(0, 1), (1, 3), (2, 5), (3, 7)}
D. f - 1(x) = {(-3,-2), (-4,-1), (-5,0), (-6,1), (-7,2)}

O O O O 5. Which of the following table of values show the correct inverse


function of the one-to-one (y = 2x + 3) function as presented in the
graph below?

f - 1(x) 3 4 5 6 7 f - 1(x) 3 1 -1 -3 -5
A. y -2 -1 0 1 2 C. y 0 -1 -2 -3 -4

f - 1(x) 1 0 -1 -2 -3 f - 1(x) 0 -1 -2 -3 -4
y 0 1 2 3 4 y 3 1 -1 -3 -5
B. D.

-----------------------------------------------------------------------------------------------------

78
Competency
Graphs the inverse function. M11GM-Ie-1

Objectives
At the end of the lesson, the learners are expected to:
1. Identify inverse function by their graphs.
2. Graph the inverse functions.

Procedure/Learning Experience
Activity
Graph the following one-to-one function and its inverse function illustrated in the
table below.
One-to-one function Inverse Function
f(x) -2 -1 0 1 2 f - 1(x) 3 4 5 6 7
y 3 4 5 6 7 y -2 -1 0 1 2
1

f(x) -4 -2 0 2 4 f - 1(x) -6 -3 0 3 6
2
y -6 -3 0 3 6 y -4 -2 0 2 4

f(x) 0 1 2 3 4 f - 1(x) 1 0 -1 -2 -3
3
y 1 0 -1 -2 -3 y 0 1 2 3 4

f(x) 7 6 5 4 3 f - 1(x) 5 3 2 1 -1
4
y 5 3 2 1 -1 y 7 6 5 4 3

f(x) -5 -3 -1 1 3 5 f - 1(x) 5 3 1 -1 -3 -5
5
y 5 3 1 -1 -3 -5 y -5 -3 -1 1 3 5

Analysis
1. How do you find the activity?
2. Examine each graph, compare the graph of one-to-one function to inverse
function?
3. What have you noticed to their graphs?
4. What can you say about the graph of an inverse function?

79
Abstraction
Guidelines for Finding the Inverse Functions by their Graphs:
1. Sketch both graphs on the same coordinate grid.
2. Draw the line y = x and look for symmetry.
a. If no symmetry is evident, the functions are not inverse functions.
b. If symmetry is apparent, go to step 3 to verify.
3. Compare the coordinates of at least four points to determine if they are
reversed. If so the functions are inverses.

Example:
Sketch the graphs of f(x) = x + 2 and f -1(x) = x - 2 for values -2 ≤ x ≤ 2 and
determine if they are inverse functions.
Complete the table below:
f(x) = x + 2 [blue] f -1(x) = x - 2 [red]
f(x) -2 -1 0 1 2 f - 1(x) -2 -1 0 1 2
y 0 1 2 3 4 y -5 -3 -2 -1 0

Step 1: Sketch
both graphs on the
same coordinate
grid.

Step 2: Draw the


line y = x and look
for symmetry.

80
As presented below,
Step 3. Compare
f(x) = x + 2
the coordinates of
at least four points f(x) -2 -1 0 1 2
to determine if y 0 1 2 3 4
they are reversed.
If so the functions If we continue the value of its inverse to 4, then the
are inverses. coordinates will then be:
f -1(x) = x - 2
f - 1(x) -2 -1 0 1 2 3 4

y -5 -3 -2 -1 0 1 2
Then, the graph is an inverse function.

Application
Show that each of the following functions are inverses.
a) f(x) = ; g(x) = b) f(x) = x-1 ; g(x) = x + 1
c) f(x) = 2x + 3; g(x) = (x - 3)/2 d) f(x) = 5x - 3; g(x) = 3 - 5x

Reflection
Remember that an inverse function is a one-to-one function. Whereas, the
domain of the inverse function is the range of the one-to-one function and the
range of the inverse function is the domain of the one-to-one function. Its graph is
symmetrical to each other.
-------------------------------------------------------------------------------------------------------------

81
Posttest
Shade the circle corresponding to the letter of the best answer.
A B C D
O O O O 1. Which of the following graphs DOES NOT belong to the group?
A. C.

B. D.

O O O O 2. Which of the following coordinates shows the correct one-to-one


(y=x2) of the inverse function [broken line] as presented in the
graph below?
A. f (x) -2 -1 0 1 2
y 4 1 0 1 4

B. f (x) 4 1 0 1 4
y -2 -1 0 1 -2

C. f (x) -2 -1 0 1 2
y -2 -1 0 1 2

f (x) 0 -1 -2 -3 -4
D.
y 3 1 -1 -3 -5

82
O O O O 3. This equation is used to test if the graph of one-to-one function is
its inverse function.
A. f(x) = y C. f(x) = x - y
B. y = x D. f(x) = x + y

O O O O 4. Describe the behavior of the graph of a one-to-one function and


its inverse.
A. Hyperbolic C. Parallel
B. Parabola D. Symmetric

O O O O 5. Which of the following ordered pairs of inverse function has its


one- to-one function in-
f (x) 2 5 8 11 14 dicated in the table
be- y -2 -1 0 1 2 low:

A. f - 1(x) = {(-2,2), (-1,5), (0,8), (1,11), (2,14)}


B. f - 1(x) ={(-2,2), (5,-1,), (8,0), (11,1), (14,2)}
C. f - 1(x) = {(-2,2), (11,5), (0,8), (11,1), (2,14)}
D. f - 1(x) = {(2,-2), (5,-1), (8,0), (-11,1), (14,2)}
-------------------------------------------------------------------------------------------------------------
End of Lesson 11

83
Lesson 12: Problems Involving Inverse Function
-------------------------------------------------------------------------------------------------------------
Pretest
Shade the circle corresponding to the letter of the best answer.
A B C D
O O O O 1. Which of the following equations is an inverse of the function
f(x) = x + 9?
A. f- 1(x) = x2 - 9 C. f- 1(x) = 9x - 1
B. f- 1(x) =x-9
-1
D. f (x) = 9x + 1

O O O O 2. From the illustration of function g(x) with its g-1(x) function, what
can you say about their coordinates?
A. They reflected through y = x.
B. They do not reflected at y = x.
C. Few of the coordinates are reflected.
D. Only one coordinate is symmetric.

O O O O 3. In converting degree temperature, what is the formula to convert


degree Fahrenheit to degree Celsius?
A. T(°F) = T(°C) × 9/5 + 32 C. T(°F) = T(°C) × 9/5 - 32
B. T(°C) = (T(°F) - 32) × 5/9 D. T(°C) = (T(°F) + 32) × 5/9

O O O O 4. What is the inverse function of f(x) = 3x - 5?


A. f- 1 (x) = 3x - 5 C. f- 1 (x) = 3x + 5
B. f- 1 (x) = x + 5 D. f- 1 (x) = x - 5
3 3
o o o
O O O O 5. If 0 C is 32 F, what is 50 F to degree Celsius?
A. 10oC C. 82oC
B. 18oC D. 160OC
-------------------------------------------------------------------------------------------------------------

84
Competency
Solves problems involving inverse function. M11GM-Ie-2

Objectives
At the end of the lesson, the learners are expected to:
1. Understand and investigate problems involving inverse function.
2. Use mathematical models or expressions to solve problems involving
inverse functions.
3. Solve real-life problems involving inverse functions.

Procedure/Learning Experience
Activity
1. Converting OCelsius to OFahrenheit: Dry ice, or frozen carbon dioxide sublimes
(phase change between solid to gas) at -83.5 OC.
A. What is this temperature in Fahrenheit?
B. What is 212OF in OC ?

2. In a bag of red and blue sweets, the ratio of red sweets to blue sweets is 3:4.
A. If the bag contains 120 blue sweets, how many red sweets are there?
B. If the bag contains 180 red sweets, how many blue sweets are there?

Analysis
1. How do you find the activity?
2. What did you notice to the process in each item?
3. Are they reversibly proportional to each other?

Abstraction
We can use mathematical models or expression and reverse the process to
apply the concepts of solving word problems involving inverse function.
Example 1. Complete the table below, converting OF to OC.
O
F 0 10 20 30 32
O
C

85
Solution:
Use T(°C) = (T(°F) - 32) × 5/9 to find OC.

If OF = 0, If OF = 10, If OF = 32,
T(°C) = (T(°F) - 32) × 5/9 T(°C) = (T(°F) - 32) × 5/9 T(°C) = (T(°F) - 32) × 5/9
T(°C) = (T(0) - 32) × 5/9 T(°C) = (T(10) - 32) × 5/9 T(°C) = (T(32) - 32) × 5/9
T(°C) = (T(-32) × 5/9 T(°C) = (T(-22) × 5/9 T(°C) = (T(0) × 5/9
T(°C) = -17.78 T(°C) = -12.22 T(°C) = 0

If OF = 20, If OF = 30,
T(°C) = (T(°F) - 32) × 5/9 T(°C) = (T(°F) - 32) × 5/9
T(°C) = (T(20) - 32) × 5/9 T(°C) = (T(30) - 32) × 5/9
T(°C) = (T(-12) × 5/9 T(°C) = (T(-2) × 5/9
T(°C) = -6.67 T(°C) = -1.11

Therefore, the inverse function of T(°C) = (T(°F) - 32) × 5/9 is


T(°F) = T(°C) × 9/5 + 32.
O
F 0 10 20 30 32
O
C -17.78 -12.22 -6.67 -1.11 0

Example 2. Engineers have determined that the maximum force t in tons that
a certain bridge can carry is related to the distance d in meters between it supports
by using the function t(d) = (12.5/d)3. How far should the supports be if the bridge is
to support 3.2 tons? Construct an inverse function to represent mathematical
equation and to determine the result.

Solution: The equation of the function is t = (12.5/d)3.


To lessen confusion in this case, let us not interchange d and t as they
denote specific values. Solve instead for d in terms of t:
t = (12.5/d) 3

d = 12.5 /
The inverse function is d(t) = 12.5 / .
Evaluate the function at t = 3.2: d(3.2) = 12.5 / = 8.48
The supports should be placed at most 8.48 meters apart.

86
Application
Solve each of the following problems.
1. Find the inverse function of .
2. Complete the table:
O
C 0 10 30 60 90
O
F

3. When calibrating a spring scale, you need to know how far the spring stretches
based on given weights. Hooke’s law states that the length a spring stretches is
proportional to the weight attached to the spring. A model for one scale is in the
equation l = 0.5w + 5 where l is the total length (in inches) of the spring and w is
the weight (in pounds) of the object.
a. Find the inverse model for the scale.
b. If you place a melon on the scale and the spring stretches to a total length of
5.5 inches, how much does the melon weigh?

Reflection
Remember that we can apply the concepts of inverse functions in solving
word problems involving reversible processes.
-------------------------------------------------------------------------------------------------------------

87
Posttest
Shade the circle corresponding to the letter of the best answer.
A B C D
O O O O 1. Which of the following equations is an inverse of the function
f(x) = 7x + 9?
A. f- 1 (x) = -7x + 9 C. f- 1 (x) = 7x - 9
B. f- 1 (x) = x + 9 D. f- 1 (x) = x - 9
7 7
O O O O 2. From the illustration of function g(x) with its g(x)-1 function, what
can you say about their
coordinates?

A. They are symmetric to each other.


B. They are not symmetric.
C. Few of the coordinates are symmetric.
D. Only one coordinate is symmetric.

O O O O 3. In converting degree temperature, what is the formula to convert


degree Celsius to degree Fahrenheit?
A. T(°F) = T(°C) × 9/5 + 32 C. T(°F) = T(°C) × 9/5 - 32
B. T(°C) = (T(°F) - 32) × 5/9 D. T(°C) = (T(°F) + 32) × 5/9

O O O O 4. What is the inverse function of f(x) = x3 - 7?


A. f- 1 (x) = C. f- 1(x) = 3x + 7
-1
B. f (x) = D. f- 1 (x) = 3x - 7

O O O O 5. If 10oC is 50oF, what is 100oF to degree Celsius?


A. 32oC C. 212oC
B. 180oC D. 320OC
-------------------------------------------------------------------------------------------------------------

88
Answer Key
Chapter 1
Lesson 1
Pretest
1. A 2.D 3.C 4.D 5. A
Activity
Group A Group B
A, D, E, G, I B, C, F, H, J
Reason: [Answer may vary] Group A has a unique element in the first component of the
ordered pair, while Group B repeats the elements in the first component of the ordered pair
(vice versa).
Application
A. B. 1. y = 2x - 1 2. h(x) =
1. Mere relation D = {x/x є Ɽ} D ={x/x ≥ 3}
2. Mere relation R = {y/y є Ɽ} R = {h(x)/h(x) ≥ 0}
3. Function
4. Mere relation
5. Function
3. m(x) = x2 + 3 4. y = 5. y = |x| + 2
D ={x/x є Ɽ} D ={x/x ≠ 3} D ={x/x є Ɽ}
R = {m(x)/m(x) ≥ 3} R = {y/y ≠ 0} R = {y/y ≥ 2}
C.
1. S(n) = 750n
2. R(t) = 15t
3. A(n) = 250n

Posttest
1. C 2. D 3. A 4. C 5. D
—————————————————————————————————————
Lesson 2
Pretest
1. A 2. C 3. A 4. C 5. A
Application
Evaluate the following functions at x = 5.
1. f(5) = 0
2. g(5) = 12
3. h(5) =
4. d(5) = 4
5. f(5) = 6
————————————————————————————————————————-

89
Find the function value.
1. f(4) = 23 and f(-5) = -40
2. f(1) = -5 and f(-3) = -9
3. f(3) = 6 and f(-2) = -1/4

a. f(x) = 35 for 0 < x ≤ 1


Let x = additional hour of parking
x - 1 = hour of parking over and above 1 hour.
f(x) = 35 + 15(x - 1)
f(x) = 35 + 15x - 15
f(x) = 35 - 15 + 15x
f(x) = 20 + 15x for x > 1
b. f(x) = 20 + 15x for x > 1
f(8) = 20 + 15(8)
f(8) = 140
c.f(x) = 20+ 15x for x > 1
8 am to 1:30 = 5.5 hours or 6 hours
f(x) = 20 + 15x for x > 1
f(6) = 20 + 15(6)
f(6) = 110
Posttest
A 2. B 3. B 4. A 5. C
—————————————————————————————————————
Lesson 3
Pretest
C 2.D 3. A 4. B 5. A
Activity
1. 9x2 +3x - 4 4.

2. 5.

3.

Application
Let and . Find the following:
2
1. = x - 2x + 8
= x2 - 4x - 8
= x3 + 5x2 - 24x
=

= x2 +13x + 40

90
A. Define the following functions and determine the domain of the resulting function.
1. Find:
Given: and
Answer: 2x - 3; domain:{x/x є Ɽ}
2. Find:
Given: and
Answer: x - 7; domain:{x/x є Ɽ}
3. Find:
Given: and
2
Answer: 2x - 2x - 12; domain:{x/x є Ɽ}
4. Find:
Given: and

Answer: ; domain:{x/x є Ɽ}

5. Find:
Given: and
Answer: 10(x - 1); domain:{x/x є Ɽ}
Posttest
1. B 2. D 3. A 4. B 5. A
——————————————————————————————————
Chapter 2
Lesson 4
Pretest
1. D 2. A 3. D 4. B 5. C

Application
Determine whether the given is a rational function, a rational equation, a rational
inequality or none of these.
1. Rational Equation 4. Rational inequality
2. None of these 5. Rational function
3. Rational function
Solve the following rational equations/inequalities.
1.
2.
3.
4. (-2,0)
5. [1,4)

91
Posttest
1. A 2. D 3. B 4. C 5. A

Lesson 5
Pre-test
1. D 2. A 3. A 4. B 5. C

Application
1. Solve the following.
Given

2. Represent the speed of a runner as a function of the time it takes to run 200
meters in the track. Construct a table of values for the speed of a runner and
sketch a graph by plotting and connecting the points.
f(x) = 200/x

3. A hypothetical function representing the concentration of a drug in a patient’s


bloodstream over time t (in hours) is given as .
1. Construct a table of values.
2. Plot the points in a Cartesian plane and connect them.
3. What can you say about the function?

92
At t = 0, the concentration is zero since the drug has not entered the
bloodstream yet. It shoots up at t = 1 but it starts decreasing after that.
Posttest
1. D 2. A 3. A 4. B 5.C

Lesson 6
Pretest
1. A 2. D 3. C 4. D 5. B

Application
x -3 -2 -1 0 1 2 3
f(x) 2 3 -1 0 0.33 0.5

Post Test
1. C 2. C 3. B 4. C 5.D

93
Lesson 7
Pretest
1. A 2. C 3. B 4. B 5. A
Application
1. x-intercept is -1 and y-intercept is 1
2. vertical asymptote at y=1 and horizontal asymptote at x=1
3. Domain= All real numbers except 1
4. Range= All real numbers except 1
5. Graph of the function

Posttest
1. B 2. D 3. A 4. C 5. B
—————————————————————————————————————
Chapter 3
Lesson 8
Pretest
1. B 2. A 3. C 4. B 5. A

Application
1. Since the dog is related to all the family members, therefore it is not one-to
one.
2. A certain passport can only belong to a certain person, therefore it is one-to
one.
3. A manufacturer produces thousands of car for a certain model, therefore it is
not one-to-one.
Posttest
1. C 2. A 3. C 4. B 5. A
——————————————————————————————————
Lesson 9
Pretest

94
Application
The functions are inverses of each other.

The two functions are not the inverse of each other since the inverse of
is

Posttest
1. B 2. C
3. D 4. C 5. B
—————————————————————————————————————

95
Lesson 10
Pretest
1. B 2. A 3. C 4. D 5. D
Activity
1. N 2. O 3. N 4. O 5. O
Application: (encourage the learner to use table of values)
1. Domain: {X Є ℝ} and Range: {Y Є ℝ} 4. Domain: {X Є ℝ} and Range: {Y Є ℝ}
2. Domain: {X Є ℝ} and Range: {Y Є ℝ} 5. Domain: {X Є ℝ} and Range: {Y Є ℝ}
3. Domain: {X Є ℝ} and Range: {Y Є ℝ}

Posttest
1. C 2. A 3. C 4. D 5. D

Lesson 11
Pretest
1. A 2. D 3. C 4. B 5. C

Application
a) f(x) = x2 – 4; g(x) = b) f(x) = x - 1 ; g(x) = x + 1
a) = –4 b.
= +4 –4 =
= =
c) f(x) = 2x + 3; g(x) = 2 d. f(x) = 2x - 1 ; g(x) = x - 1
= = =
= =
Post-test
1. C 2. D 3. B 4. A 5. D
—————————————————————————————————————

96
Lesson 12
Pretest
1. B 2. A 3. B 4. B 5. A
Application
1. The formula needed to convert Celsius to Fahrenheit is: F = 9⁄5°C + 32 where °F
and °C are the temperatures in Fahrenheit and Celsius respectively.
To solve this, plug the Celsius temperature,-83.5 °C into the formula.
°F = 9⁄5(-83.5) + 32
°F = -150.3 + 32
°F = -118.3 °F
Step 1: Assign variables:
Let x = red sweets
Write the items in the ratio as a fraction.
red = 3 = x
blue 4 120

Step 2: Solve the equation by Cross Multiply


3 × 120 = 4 × x
360 = 4x
Isolate variable x
x = 360/4
Answer: There are 90 red sweets.

B. Write the items in the ratio as a fraction.


red = 3 = 180
blue 4 x

Step 2: Solve the equation by Cross Multiply


4 × 180 = 3 × x
720 = 3x
Isolate variable x
x = 720/3
Answer: There are 240 red sweets.

Posttest
1. D 2. A 3. A 4. A 5. C

—————————————————————————————————————

97
GLOSSARY
Asymptote-a line that a graph will approach but will not attain its value

Composite Function-a function whose values are found from two given functions
by applying one function to an independent variable and then applying the
second function to the result and whose domain consists of those values of the
independent variable for which the result yielded by the first function lies in the
domain of the second.

Domain-the set of all permissible input values of a function.

Extraneous Solution-If you get a solution to a rational equation that matches


where the rational function is undefined.

Function-is a relation or rule of correspondence such that each element in the


domain corresponds to exactly one element in the range.

Horizontal Asymptote-The horizontal line y=b is a horizontal asymptote of the


function f if f(x) gets closer to b as x increases or decreases without
bound.

Horizontal Line test-A graph passes the Horizontal line test if each horizontal line
cuts the graph at most once.

Inverse Function-Two functions are said to be inverses of each other if they


are the reverse process of each other. The function that reverses
the roles of x and y is the inverse function of f and is denoted .

Interval Notation-An inequality may have infinitely many solutions. The set of all
solutions can be expressed using set notation or interval notation.

One-to-One Functions-A function f is one-to-one if it never takes the same value


twice or . That is, the same y-value is never
paired with two different x-values.

Piecewise Functions-These functions are defined compositely using several


expressions and different interval domains.

Range-the set of all output values of a function.

Rational Equation-an equation that contains rational expressions.

Rational Function-a rational function r(x), is a function in the form of ,

98
Rational Inequality-an inequality that contains rational expressions.

Relation-any set of ordered pairs.

Vertical Asymptote-The vertical line x=a is a vertical asymptote of a function f if the


graph increases or decreases without bound as the x values approach a
from the right or left.

X-Intercept-The x-intercepts of a rational function are the values of x that will make
the function zero.

Y-Intercept-The y-intercept of a function is equal to f(0)

99
References

Books
Alday, E. et. al. General Mathematics. Diwa Learning Systems Inc., 2016.

Alferez, A. et. al. Advanced Algebra. MSA Publishing House, 2008.

Department of Education.General Mathematics Learner’s Material. Lexicon Press


Inc.

Orines, B. et. Al. Advanced Algebra: Trigonometry and Statistics. Phoenix


Publishing House Inc., 2004.

Orines, F. Next Century Mathematics 11: General Mathematics. Phoenix


Publishing House, 2016.

Oronce, Orlando and Mendoza, Marilyn O. General Mathematics. Rex Bookstore,


Inc., 2016.

Oronce, Orlando, General Mathematics. Rex Bookstore, Inc. 2016.

Websites

Talcott Parsons Quotes about functions AZ QUQTEST https://www.google.com /


searchq=quotes+about+functions&source=lnms&tbm=isch&sa=X&ved=0ahUK
EwjrqKuTsrniAhVZa94KHVudDVEQ_AUIDigB#imgrc=81svKmqV_s1JKM:

https://www.google.com/search?tbm=isch&sa=1&ei=5KDqXKr-Bp-
moATQ_ZTAAw&q=quotes+about+rational&oq=quotes+about+rational&gs_l=i
mg.3..35i39j0i19l2.137973.138935..139736...0.0..0.405.2236.2-
4j1j2......0....1..gws-wiz
img.......0i5i30i19.SVAjmjBU9QY#imgrc=wF1L6Gh23nU8tM:

Malala Yousafzai Quotes About One to One https://www.google.com/search?


tbm=isch&sa=1&ei= WaLqXJeWGMTn-
QbVxoqwAw&q=quotes+about+one+to+ one+&oq=
quotes+about+one+to+one+&gs_l =img.3...91510.93090..
93922...0.0..0.0.0.......0....1..gws-wiz-
img.fmdxHlLgwpw#imgrc=BmWAjfV9u47JGM:

100
For inquiries or feedback, please write or call:

Schools Division of Lucena City


M.L. Tagarao Extension, DepEd Administrations Bldg.
Ilayang Iyam, Lucena City, Quezon Philippines 4301
Telephone No: 042-660-4472
Email Address: lucena.city@deped.gov.ph
For inquiries or feedback, please write or call:

Schools Division of Lucena City


M.L. Tagarao Extension, DepEd Administrations Bldg.
Ilayang Iyam, Lucena City, Quezon Philippines 4301
Telephone No: 042-660-4472
Email Address: lucena.city@deped.gov.ph

You might also like